Finding limits of line integral

Click For Summary
The discussion revolves around solving a line integral from (0,0) to (\pi,-1) using parameterization. The user parameterizes the line with x = \pi t and y = 1 - 2t, leading to a new integral in terms of t. They express uncertainty about changing the limits of integration from two variables to one, seeking clarification on the correct limits for t. Despite calculating the integral and obtaining a value of -1, they encounter a discrepancy when checking the independence of the path, resulting in a value of 0. The user requests assistance in identifying errors in their approach.
boneill3
Messages
126
Reaction score
0

Homework Statement



Integrate along the line segment from (0,0) to (\pi,-1)
The integral

\int_{(0,1)}^{(\pi,-1)} [y sin(x) dx - (cos(x))]dy



Homework Equations





The Attempt at a Solution



I have used the parameterization of x=\pi t and y= 1-2t
To get the integral:
\int_{(0,1)}^{(\pi,-1)} [1-2t sin(\pi t) -(cos(\pi t))]dt

But now because it is an integral of variable t I need to change the limits .

I'm not sure if I just have to put the limits of t just from 0 to \pi

I suppose I'm having trouble with getting from the limit of 2 variables (x,y) to a limit of one variable t

Thanks
 
Physics news on Phys.org
If x=pi*t and y=1-2t, then if you put t=0 then x=0 and y=1, right? If you put t=1 then x=pi and y=(-1), also right? As you came up with that fine parametrization what's the problem with finding limits for t?
 
Thank you for your help.
I will need to go back and study more about parametrization.
 
When calculating this line integral

\int_{(0,1)}^{(\pi,-1)} [y sin(x) dx - (cos(x))]dy

I'm using the formula
\int_{a}^{b}[f(x(t),y(t))x'(t) + g(x(t),y(t))y'(t)]dt

with parameterization

I have x = \pi t
y = 1-2t
so
x' = \pi
and
y' = -2

plugging into the integral I get

\int_{(0)}^{(1)} [1-2y sin(\pi t) \pi - (cos(\pi t))-2]
= -1

The question states that the integral is independant of path.

So if I integrate along the initial line segment (0,1)to (0,\pi)
I should be able to plug in the values f((-1,\pi))-f((0,1))
And it should equal my original integral vaue of -1.

However I get 0

Could someone please check what I've done I show me where I am going wrong ?
 
Question: A clock's minute hand has length 4 and its hour hand has length 3. What is the distance between the tips at the moment when it is increasing most rapidly?(Putnam Exam Question) Answer: Making assumption that both the hands moves at constant angular velocities, the answer is ## \sqrt{7} .## But don't you think this assumption is somewhat doubtful and wrong?

Similar threads

  • · Replies 9 ·
Replies
9
Views
1K
  • · Replies 2 ·
Replies
2
Views
1K
  • · Replies 3 ·
Replies
3
Views
2K
  • · Replies 5 ·
Replies
5
Views
2K
  • · Replies 2 ·
Replies
2
Views
1K
  • · Replies 6 ·
Replies
6
Views
2K
  • · Replies 19 ·
Replies
19
Views
2K
Replies
12
Views
2K
Replies
2
Views
2K
  • · Replies 4 ·
Replies
4
Views
2K